Search found 31 matches


armaan700+ wrote:it luks all ans are wrng
Armaan700+

If you explain the reason for all to be wrong,it would be easier for everybody to put their reasons.In my opinion choice A looks ok.

by ramsharma

Wed Feb 03, 2010 12:51 am
Forum: Sentence Correction
Topic: princeton test
Replies: 2
Views: 1017

@ Ron N all other Experts, Acc to me, Ans is A or C (Depending upon 'Focus' is singular or Plural) So plz confirm this issue first Secondly , as i know , Even in case of subjenctive, we omit TO from first verb (Infinitive verb form ) NOT SECOND Right ? Plz respond only if U r DAMN SURE otherwise it...

by ramsharma

Sat Jan 30, 2010 8:12 am
Forum: Sentence Correction
Topic: subject, verb
Replies: 6
Views: 1812

money9111 wrote:does a split infinitive always include an adverb?
Most of the time a split infinitive includes adverb or adverbial phrase.But some time in compound split infinitive a pronoun may be seen. e.g "I need you to all really pull your weight"

by ramsharma

Fri Jan 29, 2010 9:05 am
Forum: Sentence Correction
Topic: please explain
Replies: 8
Views: 1447

THE CALL IS B/N A AND C B IS OUT FOR WRONG IDIOM(EITHER ......OR) D AND E ARE OUT FOR using And for contrast , which is incorrect THE S/C IS TRYING TO EMPHASIS ON CONTRAST AND THERFORE BUT IS MORE APPROPRIATE THAN ALTHOUGH HENCE C C too has similar problem as B(EITHER ......OR). IMO:A Komal OA pls

by ramsharma

Tue Jan 26, 2010 9:22 am
Forum: Sentence Correction
Topic: New York's small museums
Replies: 8
Views: 1509

Ksunder is right.Try to solve with double matrix method.Please see attached documents

by ramsharma

Tue Jan 26, 2010 3:41 am
Forum: Data Sufficiency
Topic: Roman Party - Overlapping Sets
Replies: 6
Views: 1452

ramsharma, I don't get how you come from 10^k/5 to 2*10^k-1 can you explain in more details. = 10^k/5 =2*10^k/(2*5) -multiplied numerator and denominator by 2(eg 3/5=2*3/5*2) =2*10^k/10 -Denominator(2*5=10),Numerator remains as it is(eg 3/5=2*3/5*2=2*3/10) =2*10^k-1 (10^k/10=10^k-1 eg.10^11/10=10^(...

by ramsharma

Tue Jan 19, 2010 3:36 am
Forum: Problem Solving
Topic: (2^k)(5^k-1)=?
Replies: 8
Views: 3895

bhumika.k.shah wrote:sowree :(
the OA is A
how did you get the final step ? where did the 2 come from? :-P
In the final step he multiplied both numerator and denominator by 2

= 10^k/5
=2*10^k/(2*5)
=2*10^k/10
=2*10^k-1

by ramsharma

Mon Jan 18, 2010 9:42 am
Forum: Problem Solving
Topic: (2^k)(5^k-1)=?
Replies: 8
Views: 3895

to find out the area of circular region we need either BC or BO Statement 1.This gives only the ratio of BC and AB.BC will vary when ab will vary(when AB=16,BC=9 and when AB=32 ,BC=18.) hence insufficient Statement 2.This provides only BD and we can not calculate BC with this. Insufficient Statement...

by ramsharma

Sun Jan 17, 2010 8:17 am
Forum: Data Sufficiency
Topic: Geometry DS
Replies: 1
Views: 3308

Are all angles of triangle ABC smaller than 90 degrees? 1. 2AB = 3BC = 4AC 2. AC^2 + AB^2 > BC^2 OA A pls explain !!! Statement 1. Consider 2AB=3BC=4AC=12K WHERE K IS CONSTANT NOW 2AB=12K;AB=6K SIMILARLY BC=4K AND AC=3K SO AB:BC:AC=6:4:3 THIS IMPLIES ANGLE ACB IS MORE THAN 90 DEG(THIS ANGLE WOULD H...

by ramsharma

Sun Jan 17, 2010 8:01 am
Forum: Data Sufficiency
Topic: GMATClub m17 DS
Replies: 1
Views: 3003

The choice E is first to be eliminated as this is very much fragmented and does not have verb at all in the sentence.

by ramsharma

Thu Jan 14, 2010 10:21 am
Forum: Sentence Correction
Topic: Seldom
Replies: 17
Views: 3891

#1. IMO B Is x > y? 1. ax > ay If a is +ve, then x>y. Yes If a is -ve, then x<y. No Insufficient. 2. a^2 * x > a^2 * y Regardless of a as -ve or +ve, a^2 will always be positive. So, inequality sign does not change. x > y. Sufficient. #2. Can you post exact question and answer choices? #3. n * k = ...

by ramsharma

Mon Jan 11, 2010 12:58 am
Forum: Data Sufficiency
Topic: ETS Test paper 48 Question
Replies: 4
Views: 1584

Can someone spot the error here? From (1), x - y = 1/2 From (2), x/y > 1 so x > y Choose x= 1/2 and y = -1/2 to satisfy both 1 and 2. So x and y cannot be both positive? Please enlighten. What did you assume here? I'm sorry. I may have poor basic understanding but isn't x/y>1 equate to x>y? Multipl...

by ramsharma

Fri Jan 08, 2010 3:19 am
Forum: Data Sufficiency
Topic: Help with this inequality question, please
Replies: 8
Views: 1269

Statement 2 shoudln't be correct. Correct me if I am wrong - Set S - 1, 2, 3, 4, 5 Subset S - 3, 4, 5 Mean of set S - 3, mean of subset S - 4 Hi Vikas You have to consider all subset not a specific subset.And more over you are proving other way .It is given in the question that mean of the set does...

by ramsharma

Tue Jan 05, 2010 10:21 am
Forum: Data Sufficiency
Topic: Set S --- One more ;-(
Replies: 4
Views: 1756

from stmt1, we get a^3-a^2-b = 49 a(a^2-a)-b = 49 12a-b = 49 => NOT SUFFICIENT from stmt2, we get b^2-b-2 = 0 b = 2, -1 now only when b=-1 will we get an integer value of a = 4, so stmt2 is SUFFICIENT So i think thats how we get B, but without the ans i would have chosen C as well :( I would like t...

by ramsharma

Tue Jan 05, 2010 9:38 am
Forum: Data Sufficiency
Topic: Integers A and B
Replies: 3
Views: 1377

Hi hesh and caeser Both of you are considering Pam behind of Ed,Consider other way round the total will be different,Hence you will come to answer as E.If any confusion use number line and put Pam and Ed. -------------------P-----E--------------------=47 --------------E-----P--------------=35 Thanks...

by ramsharma

Tue Jan 05, 2010 6:14 am
Forum: Data Sufficiency
Topic: LOGIC
Replies: 3
Views: 1440